Gym - 101982F 扫描线+线段树】的更多相关文章

题目链接:https://codeforces.com/gym/101982/attachments 要你求覆盖奇数次的矩形面积并,每次更新时减去原先的值即可实现奇数次有效,下推时为保证线段长度不变左儿子的值为x[mid]-x[l]再减原来的值,右儿子的值为x[r]-x[mid]再减原来的值 #include<iostream> #include<algorithm> using namespace std; #define ll long long #define maxn 20…
题目链接:http://acm.hdu.edu.cn/showproblem.php?pid=3642 Time Limit: 10000/5000 MS (Java/Others) Memory Limit: 32768/32768 K (Java/Others) Problem Description Jack knows that there is a great underground treasury in a secret region. And he has a special d…
[BZOJ3958][WF2011]Mummy Madness Description 在2011年ACM-ICPC World Finals上的一次游览中,你碰到了一个埃及古墓. 不幸的是,你打开了坟墓之后,才发现这是一个坏主意:突然之间,原本空无一物的沙漠上已经爬满了暴躁的木乃伊.(如果你也沉睡几千年而突然被惊醒,你也会变得如此暴躁的.)(幸运的是,当你做完这道题的时候,你醒来了,发现你在弗罗里达的酒店里.那些木乃伊只是一场梦.) 面对这一大堆疯狂的木乃伊,你唯一的机会就是试图在他们抓到你之…
Description Ted has a new house with a huge window. In this big summer, Ted decides to decorate the window with some posters to prevent the glare outside. All things that Ted can find are rectangle posters. However, Ted is such a picky guy that in ev…
题目描述 给定一个序列A[i],每次询问l,r,求[l,r]内最长子串,使得该子串为不上升子串或不下降子串 输入 第一行n,表示A数组有多少元素接下来一行为n个整数A[i]接下来一个整数Q,表示询问数量接下来Q行,每行2个整数l,r 输出 对于每个询问,求[l,r]内最长子串,使得该子串为不上升子串或不下降子串 样例输入 91 2 3 4 5 6 5 4 351 61 72 71 95 9 样例输出 66564 题解 离线扫描线+线段树 考虑询问 $[l,r]$ ,对于选出子串的左端点i,右端点…
题目链接:点击打开链接 题目描写叙述:给定一些矩形,求这些矩形的总面积.假设有重叠.仅仅算一次 解题思路:扫描线+线段树+离散(代码从上往下扫描) 代码: #include<cstdio> #include <algorithm> #define MAXN 110 #define LL ((rt<<1)+1) #define RR ((rt<<1)+2) using namespace std; int n; struct segment{ double l…
题面传送门 首先我们把这两个贡献翻译成人话: 区间 \([l,r]\) 产生 \(p_1\) 的贡献当且仅当 \(a_l,a_r\) 分别为区间 \([l,r]\) 的最大值和次大值. 区间 \([l,r]\) 产生 \(p_2\) 的贡献当且仅当 \(a_l\) 为区间 \([l,r]\) 的最大值且 \(a_r\) 不是区间 \([l,r]\) 的次大值,或者 \(a_r\) 为区间 \([l,r]\) 的最大值且 \(a_l\) 不是区间 \([l,r]\) 的次大值. 我们考虑转化贡献体…
题面传送门 题意: 有一个 \(10^6\times 10^6\) 的地图.其中 \(m\) 个位置上有花,\(f\) 个矩形外围用栅栏围了起来.保证 \(f\) 个矩形两两之间没有公共点. \(q\) 组询问,每组询问给出两个整数 \(x,y\),求出: 从点 \((x,y)\) 出发,只能向下或向右走,不能越过栅栏,总共可以摘到多少朵花. \(0\leq m,f,q\leq 2\times 10^5\) 先考虑 \(f=0\) 的情况,那就是一个弱智的扫描线.从低往高扫,如果 \((x_i,…
题目链接:http://www.lydsy.com:808/JudgeOnline/problem.php?id=2584 题意:给出平面n个线段,任意两个线段严格不相交,且每个线段不平行于坐标轴.移走所有线段.每次移走一个线段,移n次,移走时只能竖直向下.向上或水平向左向右移走.每次移动时不能与当前还有的其他线段相交(顶点与顶点相交允许).要求解决两个问题: (1)题目给出了一种移走的序列,但是这个序列是不合法的.找出这个序列中最早的不合法的移动是第几个: (2)要求你给出一种合法的移动序列.…
Intervals 题目连接: http://codeforces.com/gym/100231/attachments Description 给你n个区间,告诉你每个区间内都有ci个数 然后你需要找一个最小的点集,使得满足这n个区间的条件 Input n 然后li,ri,ci Output 输出最小点集大小 Sample Input 5 3 7 3 8 10 3 6 8 1 1 3 1 10 11 1 Sample Output 6 Hint 题意 题解: 线段树+二分+贪心 首先我们贪心一…